LSAT and Law School Admissions Forum

Get expert LSAT preparation and law school admissions advice from PowerScore Test Preparation.

User avatar
 Dave Killoran
PowerScore Staff
  • PowerScore Staff
  • Posts: 5853
  • Joined: Mar 25, 2011
|
#78450
Complete Question Explanation
(The complete setup for this game can be found here: lsat/viewtopic.php?t=15592)

The correct answer choice is (B)

If the computer in office 4 was bought in 1988, a chain reaction results that forces the printer in office 4 to have been purchased in 1988 (from the first rule), the computer in office 3 to have been purchased in 1988 (from the third rule), the computer in office 2 to have been bought in 1989 (from the fourth rule), the printer in office 2 to have been bought in 1989 (from the first rule), and the printer in office 1 to have been bought in 1989 (from the second rule), resulting in the following solution:
PT1-Jun1991 game 2 #11 diagram 1.png
Accordingly, the correct answer choice is (B).

Note that, interestingly, the hypothetical produced in this answer choice could have been used to answer question #10.
You do not have the required permissions to view the files attached to this post.

Get the most out of your LSAT Prep Plus subscription.

Analyze and track your performance with our Testing and Analytics Package.